LSAT--Columnist

This topic has expert replies
Newbie | Next Rank: 10 Posts
Posts: 7
Joined: Thu Dec 01, 2011 10:57 am
Followed by:1 members

by nitts » Wed Feb 22, 2012 12:55 pm
A must be true

Newbie | Next Rank: 10 Posts
Posts: 2
Joined: Tue Feb 21, 2012 10:59 pm

by HARPREET KAUR » Sun Mar 11, 2012 12:04 am
thanks

Senior | Next Rank: 100 Posts
Posts: 97
Joined: Sun Jun 24, 2012 11:23 pm

by Ganesh hatwar » Wed Jul 04, 2012 2:48 am
Hurry A. got it right!!

Master | Next Rank: 500 Posts
Posts: 150
Joined: Sat Apr 28, 2012 11:19 pm
Thanked: 4 times

by mv12 » Thu Jul 05, 2012 10:50 am
+1 for A

Senior | Next Rank: 100 Posts
Posts: 58
Joined: Mon Aug 27, 2012 3:09 am
Thanked: 2 times

by akashkumar1987 » Fri Sep 07, 2012 2:47 am
I think D will be correct but majority of guys over here are saying A.

Reason
A)It says nothing bout the environmental policies.
B)Weaken
C)Out of scope of argument
D)Covers all the scenario and supports the last line of the stimulus.
"that environmental policies that excessively restrict the use of natural resources may diminish the wealth necessary to adopt and sustain the policies that brought about these improvements"
E)Saying sum-thing out of the reasoning.

Can anybody please tell if my reasoning is correct or not.

Senior | Next Rank: 100 Posts
Posts: 87
Joined: Mon May 07, 2012 7:57 am
Thanked: 1 times

by mparakala » Fri Oct 26, 2012 12:21 pm
ans: A

clearly indicates the relationship b/w use of natural resources and increase in wealth

Senior | Next Rank: 100 Posts
Posts: 87
Joined: Mon May 07, 2012 7:57 am
Thanked: 1 times

by mparakala » Fri Oct 26, 2012 12:28 pm
akashkumar1987 wrote:I think D will be correct but majority of guys over here are saying A.

Reason
A)It says nothing bout the environmental policies.
B)Weaken
C)Out of scope of argument
D)Covers all the scenario and supports the last line of the stimulus.
"that environmental policies that excessively restrict the use of natural resources may diminish the wealth necessary to adopt and sustain the policies that brought about these improvements"
E)Saying sum-thing out of the reasoning.

Can anybody please tell if my reasoning is correct or not.
D is incorrect because it prompts that the current policies should not exist. if you go back to the passage, it says that the claims of people are given credibility because they want to use "some" natural resources (without too many restrictions on the use) to get wealth and in turn sustain the current policies.
so, the people are not against the policies. they are just against too many restrictions on the use of natural resources.

Answer choice [A] is the right one- it supports the people's argument about the use of natural resources to sustain their wealth (this wealth would be used to sustain the current policies).

Hope this helps :)

Newbie | Next Rank: 10 Posts
Posts: 4
Joined: Thu Jun 02, 2011 11:14 pm

by elika_king » Thu Jan 10, 2013 6:50 am
SO is A the correct answer?

User avatar
GMAT Instructor
Posts: 2193
Joined: Mon Feb 22, 2010 6:30 pm
Location: Vermont and Boston, MA
Thanked: 1186 times
Followed by:512 members
GMAT Score:770

by David@VeritasPrep » Thu Jan 10, 2013 8:50 am
Yes, A is the correct answer.

This is an LSAT question from the December 2004 test, the first logical reasoning section, question 19. It is a regular strengthen question.

Did you have any other questions about this one?
Veritas Prep | GMAT Instructor

Veritas Prep Reviews
Save $100 off any live Veritas Prep GMAT Course

Master | Next Rank: 500 Posts
Posts: 171
Joined: Tue Jan 08, 2013 7:24 am
Thanked: 1 times

by rajeshsinghgmat » Mon Jan 21, 2013 11:31 pm
A for answer.

Junior | Next Rank: 30 Posts
Posts: 15
Joined: Fri Jun 22, 2012 12:21 am
Thanked: 1 times

by petrifiedbutstanding » Mon May 20, 2013 8:37 am
I said A. Here were my reasons:

B >> Use of advanced technology has nothing to do with the conclusion. The conclusion is more concerned with continually generating wealth for environmental programs.
C >> Again. No relation to the conclusion.
D >> The passage says nothing of the sort. It goes contrary to conclusion that there needs to be a fair usage of natural resources so as to generate wealth to fund these programs.
E >> This choice weakens the conclusion.

A >> Clearly reaffirms the conclusion by introducing information to suggest that the conclusion is correct.

Junior | Next Rank: 30 Posts
Posts: 22
Joined: Fri Apr 06, 2012 12:08 pm

by goldenpath » Mon Jul 22, 2013 7:02 am
Answer is A.


This is the conclusion: argue that environmental policies that excessively restrict the use of natural resources may diminish the wealth necessary to adopt and sustain the policies that brought about these improvements

You can actually treat this question as an assumption question. Answer A correctly filled the gap, telling us that nations' wealth are indeed coming from the use of natural resources, therefore restrict the use of it will diminish the wealth.

Senior | Next Rank: 100 Posts
Posts: 45
Joined: Fri Aug 30, 2013 7:51 pm

by ndqv » Sun Sep 01, 2013 8:04 am
Type: Strengthen
Stimulus: better environment is attributed to environmentalists and those who reject the excessive restrict of using natural resources.
Look at choices:
A: correct - policies needed to be sustained by economic growth, which is driven by natural resources.
B: out of scope - "social programs"
C: opposite - weaken the argument
D,E: rejected by the same reason as C's

In short, choose A

Senior | Next Rank: 100 Posts
Posts: 44
Joined: Fri Apr 20, 2012 1:02 pm

by justharsha » Mon Sep 09, 2013 3:24 pm
A it is

Master | Next Rank: 500 Posts
Posts: 152
Joined: Fri Apr 24, 2015 1:39 am
Location: Rourkela Odisha India
Thanked: 2 times
Followed by:3 members
GMAT Score:650

by akash singhal » Tue May 05, 2015 2:53 am
i didnt understood the passage properly
but i can deduce that:-

A,B,C are out of context eliminated
D weakens the columnist reading

Thus answer should be E

Anyone can advice anything about some good materials regarding critical reasoning section